Vacuum of spaceWhat exactly does outer space contain?What does it mean to say 'space exists?'Is it accurate...

Avoiding morning and evening handshakes

What happens if a wizard reaches level 20 but has no 3rd-level spells that they can use with the Signature Spells feature?

What is Crew Dragon approaching in this picture?

Do any poskim exempt 13-20-year-olds from Mussaf?

How can I mix up weapons for large groups of similar monsters/characters?

How to acknowledge an embarrassing job interview, now that I work directly with the interviewer?

Wanted: 5.25 floppy to usb adapter

How to mitigate "bandwagon attacking" from players?

For Loop and Sum

Why is working on the same position for more than 15 years not a red flag?

Can I become debt free or should I file for bankruptcy? How do I manage my debt and finances?

How to prepare vegetables for a sandwich that can last for several days in a fridge?

Auto Insert date into Notepad

What does a violin mute do?

Has the Isbell–Freyd criterion ever been used to check that a category is concretisable?

Should I choose Itemized or Standard deduction?

ip vs ifconfig commands pros and cons

Proof by Induction - New to proofs

Sometimes a banana is just a banana

Is the theory of the category of topological spaces computable?

Can a person refuse a presidential pardon?

Why is c4 a better move in this position?

Where is this triangular-shaped space station from?

When does coming up with an idea constitute sufficient contribution for authorship?



Vacuum of space


What exactly does outer space contain?What does it mean to say 'space exists?'Is it accurate to compare comets to clouds and rain?How do we know that there is no border at the end of “infinite” space?High quality Space-related blog/website?Could vacuum energy dim standard candles?Mass of quantum fluctuations in space?Does space not make sound, or can we just not hear it?Is it relevant to ask the limits of space?Who was the first to realize that the Earth is surrounded by vacuum?













2












$begingroup$


Firstly I’m genuinely interesting in a working explanation for this question. It is for this reason that I am editing the question to fine tune the question. In essense the question has remained the same.



In order for me to check mark the best answer I’m going to ask for citations for the claims made in the answers because I’m getting lots of theory and responders are not in agreement, as far as I can tell. Also I cannot accept circular reasoning i.e. if we assume x then y, since y therefore x. That’s fallacious logical reasoning because x was never proven it was assumed and y might be independent of x.



The vacuum of space is incredibly powerful, 1 x 10-17 torr, and the vacuum between the Earth and the Moon is 1 x 10-11 torr.



How can such a vacuum (very low pressure) in close proximity to the Earth’s atmosphere (high pressure) that goes to 8.5km elevation, coexist with the open system of the Earth’s atmosphere? How does this not defy the second law of thermodynamics and still remain true?



Space is low pressure and the earth’s atmosphere is high pressure. In order to have any pressure the gas requires (demands) that it press upon something.




Pressure is a force exerted by the substance per unit area on another substance. The pressure of a gas is the force that the gas exerts on the walls of its container. When you blow air into a balloon, the balloon expands because the pressure of air molecules is greater on the inside of the balloon than the outside. Pressure is a property which determines the direction in which mass flows. If the balloon is released, the air moves from a region of high pressure to a region of low pressure and the balloon deflates. 1




Earth is an open system that presses against the vacuum of space, why therefore is the second law of thermodynamics suspended if indeed it is a law.
Space is low pressure, therefore the atmosphere which is not in a container should disperse into the low pressure space.




  • VACUUM (There’s nothing to it…)


  • Vacuum Technology PHY451 October 22, 2014











share|improve this question









New contributor




Autodidact is a new contributor to this site. Take care in asking for clarification, commenting, and answering.
Check out our Code of Conduct.







$endgroup$








  • 1




    $begingroup$
    I've voted to close for off-topic because... This question does not appear to be about astronomy, within the scope defined in the help center. This has become clear in a series of comments 1, 2 This is about the physics of the Earth's atmosphere (paraphrasing: why they have them, how gravity keeps them there even though vacuum is pulling at them) not astronomy
    $endgroup$
    – uhoh
    17 hours ago








  • 1




    $begingroup$
    You shouldn't change a question significantly after an answer has been posted. This is not a forum. The proper way is to post a new question.
    $endgroup$
    – Peter Mortensen
    16 hours ago










  • $begingroup$
    I can't really understand, how do you understand "incredibly powerful" to a $10^{-17}$ torr rare gas. Please fix it asap.
    $endgroup$
    – peterh
    14 hours ago










  • $begingroup$
    I didn’t realize that the perception of space (astronomy related) would end up avoiding the question altogether @Mortensen on a physics technicality, the very argument being used to close down the question. The question was about the earth next to the vacuum of space, what the value of the vacuum is, 10^-17 or 10^-11 or other is not the question, it’s about positive pressure in an open system, not a container (tube) and negative pressure of a vacuum. If this is not astronomy related then why attempt to respond in the first place. My question was not changed in essense
    $endgroup$
    – Autodidact
    14 hours ago






  • 1




    $begingroup$
    I see, the second law of thermodynamics would demand that the rare gas which has a torr value reach equilibrium with the atmosphere immediately next to it, which has a higher torr value. My question is why can they remain in proximity and maintain their torr values thereby defying the second law of thermodynamics. I don’t have the answer which is why I asked. Pressure demands a gas fill the space. Yet it stops at that tangential point but only partially. Why?
    $endgroup$
    – Autodidact
    13 hours ago


















2












$begingroup$


Firstly I’m genuinely interesting in a working explanation for this question. It is for this reason that I am editing the question to fine tune the question. In essense the question has remained the same.



In order for me to check mark the best answer I’m going to ask for citations for the claims made in the answers because I’m getting lots of theory and responders are not in agreement, as far as I can tell. Also I cannot accept circular reasoning i.e. if we assume x then y, since y therefore x. That’s fallacious logical reasoning because x was never proven it was assumed and y might be independent of x.



The vacuum of space is incredibly powerful, 1 x 10-17 torr, and the vacuum between the Earth and the Moon is 1 x 10-11 torr.



How can such a vacuum (very low pressure) in close proximity to the Earth’s atmosphere (high pressure) that goes to 8.5km elevation, coexist with the open system of the Earth’s atmosphere? How does this not defy the second law of thermodynamics and still remain true?



Space is low pressure and the earth’s atmosphere is high pressure. In order to have any pressure the gas requires (demands) that it press upon something.




Pressure is a force exerted by the substance per unit area on another substance. The pressure of a gas is the force that the gas exerts on the walls of its container. When you blow air into a balloon, the balloon expands because the pressure of air molecules is greater on the inside of the balloon than the outside. Pressure is a property which determines the direction in which mass flows. If the balloon is released, the air moves from a region of high pressure to a region of low pressure and the balloon deflates. 1




Earth is an open system that presses against the vacuum of space, why therefore is the second law of thermodynamics suspended if indeed it is a law.
Space is low pressure, therefore the atmosphere which is not in a container should disperse into the low pressure space.




  • VACUUM (There’s nothing to it…)


  • Vacuum Technology PHY451 October 22, 2014











share|improve this question









New contributor




Autodidact is a new contributor to this site. Take care in asking for clarification, commenting, and answering.
Check out our Code of Conduct.







$endgroup$








  • 1




    $begingroup$
    I've voted to close for off-topic because... This question does not appear to be about astronomy, within the scope defined in the help center. This has become clear in a series of comments 1, 2 This is about the physics of the Earth's atmosphere (paraphrasing: why they have them, how gravity keeps them there even though vacuum is pulling at them) not astronomy
    $endgroup$
    – uhoh
    17 hours ago








  • 1




    $begingroup$
    You shouldn't change a question significantly after an answer has been posted. This is not a forum. The proper way is to post a new question.
    $endgroup$
    – Peter Mortensen
    16 hours ago










  • $begingroup$
    I can't really understand, how do you understand "incredibly powerful" to a $10^{-17}$ torr rare gas. Please fix it asap.
    $endgroup$
    – peterh
    14 hours ago










  • $begingroup$
    I didn’t realize that the perception of space (astronomy related) would end up avoiding the question altogether @Mortensen on a physics technicality, the very argument being used to close down the question. The question was about the earth next to the vacuum of space, what the value of the vacuum is, 10^-17 or 10^-11 or other is not the question, it’s about positive pressure in an open system, not a container (tube) and negative pressure of a vacuum. If this is not astronomy related then why attempt to respond in the first place. My question was not changed in essense
    $endgroup$
    – Autodidact
    14 hours ago






  • 1




    $begingroup$
    I see, the second law of thermodynamics would demand that the rare gas which has a torr value reach equilibrium with the atmosphere immediately next to it, which has a higher torr value. My question is why can they remain in proximity and maintain their torr values thereby defying the second law of thermodynamics. I don’t have the answer which is why I asked. Pressure demands a gas fill the space. Yet it stops at that tangential point but only partially. Why?
    $endgroup$
    – Autodidact
    13 hours ago
















2












2








2





$begingroup$


Firstly I’m genuinely interesting in a working explanation for this question. It is for this reason that I am editing the question to fine tune the question. In essense the question has remained the same.



In order for me to check mark the best answer I’m going to ask for citations for the claims made in the answers because I’m getting lots of theory and responders are not in agreement, as far as I can tell. Also I cannot accept circular reasoning i.e. if we assume x then y, since y therefore x. That’s fallacious logical reasoning because x was never proven it was assumed and y might be independent of x.



The vacuum of space is incredibly powerful, 1 x 10-17 torr, and the vacuum between the Earth and the Moon is 1 x 10-11 torr.



How can such a vacuum (very low pressure) in close proximity to the Earth’s atmosphere (high pressure) that goes to 8.5km elevation, coexist with the open system of the Earth’s atmosphere? How does this not defy the second law of thermodynamics and still remain true?



Space is low pressure and the earth’s atmosphere is high pressure. In order to have any pressure the gas requires (demands) that it press upon something.




Pressure is a force exerted by the substance per unit area on another substance. The pressure of a gas is the force that the gas exerts on the walls of its container. When you blow air into a balloon, the balloon expands because the pressure of air molecules is greater on the inside of the balloon than the outside. Pressure is a property which determines the direction in which mass flows. If the balloon is released, the air moves from a region of high pressure to a region of low pressure and the balloon deflates. 1




Earth is an open system that presses against the vacuum of space, why therefore is the second law of thermodynamics suspended if indeed it is a law.
Space is low pressure, therefore the atmosphere which is not in a container should disperse into the low pressure space.




  • VACUUM (There’s nothing to it…)


  • Vacuum Technology PHY451 October 22, 2014











share|improve this question









New contributor




Autodidact is a new contributor to this site. Take care in asking for clarification, commenting, and answering.
Check out our Code of Conduct.







$endgroup$




Firstly I’m genuinely interesting in a working explanation for this question. It is for this reason that I am editing the question to fine tune the question. In essense the question has remained the same.



In order for me to check mark the best answer I’m going to ask for citations for the claims made in the answers because I’m getting lots of theory and responders are not in agreement, as far as I can tell. Also I cannot accept circular reasoning i.e. if we assume x then y, since y therefore x. That’s fallacious logical reasoning because x was never proven it was assumed and y might be independent of x.



The vacuum of space is incredibly powerful, 1 x 10-17 torr, and the vacuum between the Earth and the Moon is 1 x 10-11 torr.



How can such a vacuum (very low pressure) in close proximity to the Earth’s atmosphere (high pressure) that goes to 8.5km elevation, coexist with the open system of the Earth’s atmosphere? How does this not defy the second law of thermodynamics and still remain true?



Space is low pressure and the earth’s atmosphere is high pressure. In order to have any pressure the gas requires (demands) that it press upon something.




Pressure is a force exerted by the substance per unit area on another substance. The pressure of a gas is the force that the gas exerts on the walls of its container. When you blow air into a balloon, the balloon expands because the pressure of air molecules is greater on the inside of the balloon than the outside. Pressure is a property which determines the direction in which mass flows. If the balloon is released, the air moves from a region of high pressure to a region of low pressure and the balloon deflates. 1




Earth is an open system that presses against the vacuum of space, why therefore is the second law of thermodynamics suspended if indeed it is a law.
Space is low pressure, therefore the atmosphere which is not in a container should disperse into the low pressure space.




  • VACUUM (There’s nothing to it…)


  • Vacuum Technology PHY451 October 22, 2014








cosmology atmosphere space vacuum






share|improve this question









New contributor




Autodidact is a new contributor to this site. Take care in asking for clarification, commenting, and answering.
Check out our Code of Conduct.











share|improve this question









New contributor




Autodidact is a new contributor to this site. Take care in asking for clarification, commenting, and answering.
Check out our Code of Conduct.









share|improve this question




share|improve this question








edited 6 hours ago







Autodidact













New contributor




Autodidact is a new contributor to this site. Take care in asking for clarification, commenting, and answering.
Check out our Code of Conduct.









asked 21 hours ago









AutodidactAutodidact

1144




1144




New contributor




Autodidact is a new contributor to this site. Take care in asking for clarification, commenting, and answering.
Check out our Code of Conduct.





New contributor





Autodidact is a new contributor to this site. Take care in asking for clarification, commenting, and answering.
Check out our Code of Conduct.






Autodidact is a new contributor to this site. Take care in asking for clarification, commenting, and answering.
Check out our Code of Conduct.








  • 1




    $begingroup$
    I've voted to close for off-topic because... This question does not appear to be about astronomy, within the scope defined in the help center. This has become clear in a series of comments 1, 2 This is about the physics of the Earth's atmosphere (paraphrasing: why they have them, how gravity keeps them there even though vacuum is pulling at them) not astronomy
    $endgroup$
    – uhoh
    17 hours ago








  • 1




    $begingroup$
    You shouldn't change a question significantly after an answer has been posted. This is not a forum. The proper way is to post a new question.
    $endgroup$
    – Peter Mortensen
    16 hours ago










  • $begingroup$
    I can't really understand, how do you understand "incredibly powerful" to a $10^{-17}$ torr rare gas. Please fix it asap.
    $endgroup$
    – peterh
    14 hours ago










  • $begingroup$
    I didn’t realize that the perception of space (astronomy related) would end up avoiding the question altogether @Mortensen on a physics technicality, the very argument being used to close down the question. The question was about the earth next to the vacuum of space, what the value of the vacuum is, 10^-17 or 10^-11 or other is not the question, it’s about positive pressure in an open system, not a container (tube) and negative pressure of a vacuum. If this is not astronomy related then why attempt to respond in the first place. My question was not changed in essense
    $endgroup$
    – Autodidact
    14 hours ago






  • 1




    $begingroup$
    I see, the second law of thermodynamics would demand that the rare gas which has a torr value reach equilibrium with the atmosphere immediately next to it, which has a higher torr value. My question is why can they remain in proximity and maintain their torr values thereby defying the second law of thermodynamics. I don’t have the answer which is why I asked. Pressure demands a gas fill the space. Yet it stops at that tangential point but only partially. Why?
    $endgroup$
    – Autodidact
    13 hours ago
















  • 1




    $begingroup$
    I've voted to close for off-topic because... This question does not appear to be about astronomy, within the scope defined in the help center. This has become clear in a series of comments 1, 2 This is about the physics of the Earth's atmosphere (paraphrasing: why they have them, how gravity keeps them there even though vacuum is pulling at them) not astronomy
    $endgroup$
    – uhoh
    17 hours ago








  • 1




    $begingroup$
    You shouldn't change a question significantly after an answer has been posted. This is not a forum. The proper way is to post a new question.
    $endgroup$
    – Peter Mortensen
    16 hours ago










  • $begingroup$
    I can't really understand, how do you understand "incredibly powerful" to a $10^{-17}$ torr rare gas. Please fix it asap.
    $endgroup$
    – peterh
    14 hours ago










  • $begingroup$
    I didn’t realize that the perception of space (astronomy related) would end up avoiding the question altogether @Mortensen on a physics technicality, the very argument being used to close down the question. The question was about the earth next to the vacuum of space, what the value of the vacuum is, 10^-17 or 10^-11 or other is not the question, it’s about positive pressure in an open system, not a container (tube) and negative pressure of a vacuum. If this is not astronomy related then why attempt to respond in the first place. My question was not changed in essense
    $endgroup$
    – Autodidact
    14 hours ago






  • 1




    $begingroup$
    I see, the second law of thermodynamics would demand that the rare gas which has a torr value reach equilibrium with the atmosphere immediately next to it, which has a higher torr value. My question is why can they remain in proximity and maintain their torr values thereby defying the second law of thermodynamics. I don’t have the answer which is why I asked. Pressure demands a gas fill the space. Yet it stops at that tangential point but only partially. Why?
    $endgroup$
    – Autodidact
    13 hours ago










1




1




$begingroup$
I've voted to close for off-topic because... This question does not appear to be about astronomy, within the scope defined in the help center. This has become clear in a series of comments 1, 2 This is about the physics of the Earth's atmosphere (paraphrasing: why they have them, how gravity keeps them there even though vacuum is pulling at them) not astronomy
$endgroup$
– uhoh
17 hours ago






$begingroup$
I've voted to close for off-topic because... This question does not appear to be about astronomy, within the scope defined in the help center. This has become clear in a series of comments 1, 2 This is about the physics of the Earth's atmosphere (paraphrasing: why they have them, how gravity keeps them there even though vacuum is pulling at them) not astronomy
$endgroup$
– uhoh
17 hours ago






1




1




$begingroup$
You shouldn't change a question significantly after an answer has been posted. This is not a forum. The proper way is to post a new question.
$endgroup$
– Peter Mortensen
16 hours ago




$begingroup$
You shouldn't change a question significantly after an answer has been posted. This is not a forum. The proper way is to post a new question.
$endgroup$
– Peter Mortensen
16 hours ago












$begingroup$
I can't really understand, how do you understand "incredibly powerful" to a $10^{-17}$ torr rare gas. Please fix it asap.
$endgroup$
– peterh
14 hours ago




$begingroup$
I can't really understand, how do you understand "incredibly powerful" to a $10^{-17}$ torr rare gas. Please fix it asap.
$endgroup$
– peterh
14 hours ago












$begingroup$
I didn’t realize that the perception of space (astronomy related) would end up avoiding the question altogether @Mortensen on a physics technicality, the very argument being used to close down the question. The question was about the earth next to the vacuum of space, what the value of the vacuum is, 10^-17 or 10^-11 or other is not the question, it’s about positive pressure in an open system, not a container (tube) and negative pressure of a vacuum. If this is not astronomy related then why attempt to respond in the first place. My question was not changed in essense
$endgroup$
– Autodidact
14 hours ago




$begingroup$
I didn’t realize that the perception of space (astronomy related) would end up avoiding the question altogether @Mortensen on a physics technicality, the very argument being used to close down the question. The question was about the earth next to the vacuum of space, what the value of the vacuum is, 10^-17 or 10^-11 or other is not the question, it’s about positive pressure in an open system, not a container (tube) and negative pressure of a vacuum. If this is not astronomy related then why attempt to respond in the first place. My question was not changed in essense
$endgroup$
– Autodidact
14 hours ago




1




1




$begingroup$
I see, the second law of thermodynamics would demand that the rare gas which has a torr value reach equilibrium with the atmosphere immediately next to it, which has a higher torr value. My question is why can they remain in proximity and maintain their torr values thereby defying the second law of thermodynamics. I don’t have the answer which is why I asked. Pressure demands a gas fill the space. Yet it stops at that tangential point but only partially. Why?
$endgroup$
– Autodidact
13 hours ago






$begingroup$
I see, the second law of thermodynamics would demand that the rare gas which has a torr value reach equilibrium with the atmosphere immediately next to it, which has a higher torr value. My question is why can they remain in proximity and maintain their torr values thereby defying the second law of thermodynamics. I don’t have the answer which is why I asked. Pressure demands a gas fill the space. Yet it stops at that tangential point but only partially. Why?
$endgroup$
– Autodidact
13 hours ago












4 Answers
4






active

oldest

votes


















5












$begingroup$

update: This answer was written before the question was modified. I've tried to explain where a value like 10-17 Torr for deep space might come from, but it's since been dropped in lieu of 10-11 Torr at the Moon, which is probably a better way to formulate the question.



I think the answer is the same, two points very far away can have very different pressures. They can coexist in the same solar system, just not right next to each other. I think "Why doesn't the Moon have at least a small atmosphere?" could also be an excellent, but very different question.





In a comment the OP links to the presentation VACUUM (There’s nothing to it… ) written from the perspective of an engineer in the semiconductor manufacturing industry.



Slide 6 gives examples of vacuum levels in different situations:



Going down:




  • Low vacuum: 760 Torr to 1 x 10-3 Torr


    • Vacuum cleaner: to 600 Torr

    • Thermos bottle 10-3 Torr



  • High vacuum: 10-3 to 10-9 Torr


    • Electron microscope

    • Ion Implanter
      – Evaporator
      – Sputterer



  • Ultra high vacuum: 10-9 to 10-12 Torr


    • CERN LHC: 1 x 10-10 Torr

    • Moon’s surface: 1 x 10-11 Torr

    • Deep Space 1 x 10-17 Torr = 0.000,000,000,000,000,01 Torr




So we can see that the value of 1 x 10-17 Torr is associated with a place in "Deep Space" which is (probably) beyond that of the Moon.



Let's see if we can figure out where the author is getting that number.



According to the Wikipedia article on the interstellar medium (space between stars, far away from solar systems and other things):




In all phases, the interstellar medium is extremely tenuous by terrestrial standards. In cool, dense regions of the ISM, matter is primarily in molecular form, and reaches number densities of 106 molecules per cm3 (1 million molecules per cm3). In hot, diffuse regions of the ISM, matter is primarily ionized, and the density may be as low as 10−4 ions per cm3. Compare this with a number density of roughly 1019 molecules per cm3 for air at sea level, and 1010 molecules per cm3 (10 billion molecules per cm3) for a laboratory high-vacuum chamber.




It is harder to talk about pressure than density because pressure is related to both number density and temperature. The atmosphere is more than 10x hotter than interstellar medium, so let's look for a number density ratio of 1/10 of 1000 Torr versus 10-17 Torr, or a ratio of 1019.



If (according to Wikipedia) Earth's atmosphere has a density of 1019 per cm3, we're looking for a density of 1 per cm3. Checking Wikipedia we can see that there are components of the interstellar medium with number densities between 106 and 10-4.



It looks like the value in the presentation a rough ballpark estimate, but isn't off by more than a handful of orders of magnitude ;-)




How can such a vacuum coexist with the open system of earth’s atmosphere whereby debris from space can enter in?




While these two pressures can coexist in the same universe, they don't coexist in proximity at all. The interstellar medium is very, very far away from Earth's atmosphere, on the order of a lightyear.



Gravity keeps Earth's atmosphere nearby Earth, the solar system has gas produced by (and attracted by) the Sun's gravity. In interstellar space, there just isn't any source of gas, and what might have been there at one time has moved away, towards sources of gravity, over billions of years.






share|improve this answer











$endgroup$









  • 1




    $begingroup$
    @Autodidact You may find several questions and answers like that in those sites already, best to read them before asking something that's been answered. Here is some related math, but there are probably better places to find a useful and head-wrappable explanation. en.wikipedia.org/wiki/Scale_height
    $endgroup$
    – uhoh
    20 hours ago








  • 1




    $begingroup$
    Thank you for the link. If this question is not asked in the correct stack, my sincerest apologies. Also my edit was not intended as a moving target, I didn’t want it dismissed. As for the atmosphere pressure, while the link provides a distance it doesn’t explain my inquiry, what is it pressing upon to generate the distance? That is a question to which I cannot seem to find an answer to. Gravity seems to be the answer but it’s powers are conflicting as I’ve pointed out. It’s sufficiently strong to resist a vacuum of 10^-11 but weak enough to let gas particles up 8.5 km up into the atmosphere.
    $endgroup$
    – Autodidact
    19 hours ago






  • 1




    $begingroup$
    The Moon kind of does have an atmosphere, but it's very thin, and its total mass is under 10 tonnes. See en.wikipedia.org/wiki/Atmosphere_of_the_Moon
    $endgroup$
    – PM 2Ring
    19 hours ago






  • 1




    $begingroup$
    From the list of FAQ for Stack Exchange sites one can see for example How do comment @replies work?
    $endgroup$
    – uhoh
    17 hours ago






  • 2




    $begingroup$
    Sorry, I didn't expect the discussion to get so long. But oddly, the "Would you like this to be moved to chat" thingy still hasn't appeared.
    $endgroup$
    – PM 2Ring
    10 hours ago





















4












$begingroup$

The underlying reason that the molecules of Earth's atmosphere do not fly away into the surrounding vacuum is that they are slower than the escape velocity, which would be 11200 m/s. The typical molecule speed at ground level and room temperature appears to be 500 m/s. If it had a free path such a molecule could fly vertically for ~50s before starting to fall back, with an average velocity of 250 m/s, thus reaching an altitude of 12 or 13 km. (In reality it would collide with other molecules on the way, transferring some kinetic energy to them, so that they could in turn rise higher. Obviously, molecules at the outer fringes of the atmosphere are the real escape candidates.)



Molecules which are fast enough surely do escape Earth's gravity well. Some may have been accelerated by particles of the solar wind, some may just have been on the long tail of the standard distribution. The latter is more likely for light atoms and molecules which are faster, like Helium and Hydrogen. Hydrogen's average speed at room temperature is perhaps 2000 m/s. These gases have indeed mostly left Earth for good long ago.



(By the way, the solar wind would likely "blow away" our atmosphere in the long run — as it did on Mars — if it weren't deflected by the Earth's magnetic field.)






share|improve this answer











$endgroup$





















    1












    $begingroup$

    I think uhoh covered proximity but just to induce equilibrium to further elaborate:

    First of all, positive pressure and negative pressure are just terminology based on where we started i.e. 1 atm and above/below, we just went along. There is zero pressure and gradually moving up from that. Say, perfect vacuum starts at zero pressure and move up as comes across high pressure open system. Things are always at equilibrium and if you want to move something from low pressure to high, you do some work as you have mentioned second law of thermodynamics. Gravity does that work in this case till some point. Ignore everything, lets say there is one good looking blue dot aka earth, and as you move close, gravity starts to get stronger. So, it will invite more molecule to be cuddly and at same time pressure gradient will move gas the other way. Eventually they will reach at equilibrium i.e. same transference. This is true for any height from earth. In absence of such equilibrium earth would loose its atmosphere or gain more (may be case at astronomical time scale). We start at one (gravity rules) to continuously (important) merge to space vacuum with equilibrium all along the line. Once gas truly escape earth gravity (i.e. temperature movement is way stronger than earth gravity), it has no reason to hang around. Same way we can gain by unsuspecting wandering gas molecules. Earth atmosphere looks at equilibrium at human timescale. But earth loose and gain as these forces continuously change with distance.






    share|improve this answer








    New contributor




    HR04375439 is a new contributor to this site. Take care in asking for clarification, commenting, and answering.
    Check out our Code of Conduct.






    $endgroup$













    • $begingroup$
      At this point I really neeed citations to decide on the best answer. Could you cite your response or are you merely theorizing?
      $endgroup$
      – Autodidact
      6 hours ago



















    0












    $begingroup$

    Your assertion that our atmosphere doesn't escape is wrong.



    Helium and Hydrogen atoms have a low enough mass that they do have an escape velocity at the temperatures on the edge of our atmosphere. This means that when those gasses are released, if they fail to react on their way out, then they will be lost forever to the planet. This is why when you look at our atmosphere we just don't have any.



    You also seem to assert that there's a "line" where it's the high pressure of our atmosphere on one side, and a low pressure of space on the other; that line doesn't exist. The pressure is a gradient, in the same way as you swim down in a pool, at the top the pressure is low; at the bottom it's high; and it changes steadily as you traverse between the two. Hence why climbers of Everest need to carry oxygen.



    The heavier gasses don't escape for the same reason that a rock you throw doesn't escape. It requires energy to escape the gravitational well of Earth; and they don't have it; currently. Of course, as the atmosphere heats up from global warming, heavier and heavier particles will gain the energy to leave our atmosphere...






    share|improve this answer








    New contributor




    UKMonkey is a new contributor to this site. Take care in asking for clarification, commenting, and answering.
    Check out our Code of Conduct.






    $endgroup$













    • $begingroup$
      Ok, please cite research papers on the gradient you speak of. Also please cite how the second law of thermodynamics is suspended because of insufficient energy especially when there is no “line”. And also how pressure is maintained without a container in the open system of earth’s atmosphere. Cite whatever you think is correct but please cite it.
      $endgroup$
      – Autodidact
      5 hours ago












    • $begingroup$
      @Autodidact Why do you need citations for the pressure gradients? Surely you can take it as read? And the pressure is maintained "in the open system of the Earth's atmosphere" by gravity, as already explained. You could search for "baratropic law" or "atmosphere scale height" for more information on that.
      $endgroup$
      – Jim421616
      4 hours ago












    • $begingroup$
      Because he made an assertion and I want it backed up with science. “You also seem to assert that there's a "line" where it's the high pressure of our atmosphere on one side, and a low pressure of space on the other; that line doesn't exist. The pressure is a gradient“. That’s why I require it. Secondly I didn’t claim a line, I’m asking how this high pressure doesn’t adhere to the second law of thermodynamics, for such a gradient to even be possible in the first place. Because I’m getting gravity, mixed with buoyancy, density, mixed with pressure, gradients and stratification. Which is it?
      $endgroup$
      – Autodidact
      4 hours ago












    • $begingroup$
      @Autodidact I wouldn't start with reading papers. I get the strong impression that your understanding of the basics isn't sufficient to go through a paper and make sense of it. I would suggest just reading en.wikipedia.org/wiki/Atmospheric_pressure Consider it this way. At sea level, you have all the atmosphere above you pushing down. There's a lot of air above you, it can push down really quite hard. If you go up 500m, you have that much less air being pulled down by gravity onto you; and thus the pressure drops.
      $endgroup$
      – UKMonkey
      2 hours ago










    • $begingroup$
      @Autodidact Your comment about "buoyancy" suggests to me you're watching flat earth videos. They have a very very poor understanding of physics; and are often intentionally misleading. You may find it better to take a balloon to a swimming pool, fill it with air, and then measure the size of it at different depths to demonstrate the pressure gradient.
      $endgroup$
      – UKMonkey
      2 hours ago











    Your Answer





    StackExchange.ifUsing("editor", function () {
    return StackExchange.using("mathjaxEditing", function () {
    StackExchange.MarkdownEditor.creationCallbacks.add(function (editor, postfix) {
    StackExchange.mathjaxEditing.prepareWmdForMathJax(editor, postfix, [["$", "$"], ["\\(","\\)"]]);
    });
    });
    }, "mathjax-editing");

    StackExchange.ready(function() {
    var channelOptions = {
    tags: "".split(" "),
    id: "514"
    };
    initTagRenderer("".split(" "), "".split(" "), channelOptions);

    StackExchange.using("externalEditor", function() {
    // Have to fire editor after snippets, if snippets enabled
    if (StackExchange.settings.snippets.snippetsEnabled) {
    StackExchange.using("snippets", function() {
    createEditor();
    });
    }
    else {
    createEditor();
    }
    });

    function createEditor() {
    StackExchange.prepareEditor({
    heartbeatType: 'answer',
    autoActivateHeartbeat: false,
    convertImagesToLinks: false,
    noModals: true,
    showLowRepImageUploadWarning: true,
    reputationToPostImages: null,
    bindNavPrevention: true,
    postfix: "",
    imageUploader: {
    brandingHtml: "Powered by u003ca class="icon-imgur-white" href="https://imgur.com/"u003eu003c/au003e",
    contentPolicyHtml: "User contributions licensed under u003ca href="https://creativecommons.org/licenses/by-sa/3.0/"u003ecc by-sa 3.0 with attribution requiredu003c/au003e u003ca href="https://stackoverflow.com/legal/content-policy"u003e(content policy)u003c/au003e",
    allowUrls: true
    },
    noCode: true, onDemand: true,
    discardSelector: ".discard-answer"
    ,immediatelyShowMarkdownHelp:true
    });


    }
    });






    Autodidact is a new contributor. Be nice, and check out our Code of Conduct.










    draft saved

    draft discarded


















    StackExchange.ready(
    function () {
    StackExchange.openid.initPostLogin('.new-post-login', 'https%3a%2f%2fastronomy.stackexchange.com%2fquestions%2f29832%2fvacuum-of-space%23new-answer', 'question_page');
    }
    );

    Post as a guest















    Required, but never shown

























    4 Answers
    4






    active

    oldest

    votes








    4 Answers
    4






    active

    oldest

    votes









    active

    oldest

    votes






    active

    oldest

    votes









    5












    $begingroup$

    update: This answer was written before the question was modified. I've tried to explain where a value like 10-17 Torr for deep space might come from, but it's since been dropped in lieu of 10-11 Torr at the Moon, which is probably a better way to formulate the question.



    I think the answer is the same, two points very far away can have very different pressures. They can coexist in the same solar system, just not right next to each other. I think "Why doesn't the Moon have at least a small atmosphere?" could also be an excellent, but very different question.





    In a comment the OP links to the presentation VACUUM (There’s nothing to it… ) written from the perspective of an engineer in the semiconductor manufacturing industry.



    Slide 6 gives examples of vacuum levels in different situations:



    Going down:




    • Low vacuum: 760 Torr to 1 x 10-3 Torr


      • Vacuum cleaner: to 600 Torr

      • Thermos bottle 10-3 Torr



    • High vacuum: 10-3 to 10-9 Torr


      • Electron microscope

      • Ion Implanter
        – Evaporator
        – Sputterer



    • Ultra high vacuum: 10-9 to 10-12 Torr


      • CERN LHC: 1 x 10-10 Torr

      • Moon’s surface: 1 x 10-11 Torr

      • Deep Space 1 x 10-17 Torr = 0.000,000,000,000,000,01 Torr




    So we can see that the value of 1 x 10-17 Torr is associated with a place in "Deep Space" which is (probably) beyond that of the Moon.



    Let's see if we can figure out where the author is getting that number.



    According to the Wikipedia article on the interstellar medium (space between stars, far away from solar systems and other things):




    In all phases, the interstellar medium is extremely tenuous by terrestrial standards. In cool, dense regions of the ISM, matter is primarily in molecular form, and reaches number densities of 106 molecules per cm3 (1 million molecules per cm3). In hot, diffuse regions of the ISM, matter is primarily ionized, and the density may be as low as 10−4 ions per cm3. Compare this with a number density of roughly 1019 molecules per cm3 for air at sea level, and 1010 molecules per cm3 (10 billion molecules per cm3) for a laboratory high-vacuum chamber.




    It is harder to talk about pressure than density because pressure is related to both number density and temperature. The atmosphere is more than 10x hotter than interstellar medium, so let's look for a number density ratio of 1/10 of 1000 Torr versus 10-17 Torr, or a ratio of 1019.



    If (according to Wikipedia) Earth's atmosphere has a density of 1019 per cm3, we're looking for a density of 1 per cm3. Checking Wikipedia we can see that there are components of the interstellar medium with number densities between 106 and 10-4.



    It looks like the value in the presentation a rough ballpark estimate, but isn't off by more than a handful of orders of magnitude ;-)




    How can such a vacuum coexist with the open system of earth’s atmosphere whereby debris from space can enter in?




    While these two pressures can coexist in the same universe, they don't coexist in proximity at all. The interstellar medium is very, very far away from Earth's atmosphere, on the order of a lightyear.



    Gravity keeps Earth's atmosphere nearby Earth, the solar system has gas produced by (and attracted by) the Sun's gravity. In interstellar space, there just isn't any source of gas, and what might have been there at one time has moved away, towards sources of gravity, over billions of years.






    share|improve this answer











    $endgroup$









    • 1




      $begingroup$
      @Autodidact You may find several questions and answers like that in those sites already, best to read them before asking something that's been answered. Here is some related math, but there are probably better places to find a useful and head-wrappable explanation. en.wikipedia.org/wiki/Scale_height
      $endgroup$
      – uhoh
      20 hours ago








    • 1




      $begingroup$
      Thank you for the link. If this question is not asked in the correct stack, my sincerest apologies. Also my edit was not intended as a moving target, I didn’t want it dismissed. As for the atmosphere pressure, while the link provides a distance it doesn’t explain my inquiry, what is it pressing upon to generate the distance? That is a question to which I cannot seem to find an answer to. Gravity seems to be the answer but it’s powers are conflicting as I’ve pointed out. It’s sufficiently strong to resist a vacuum of 10^-11 but weak enough to let gas particles up 8.5 km up into the atmosphere.
      $endgroup$
      – Autodidact
      19 hours ago






    • 1




      $begingroup$
      The Moon kind of does have an atmosphere, but it's very thin, and its total mass is under 10 tonnes. See en.wikipedia.org/wiki/Atmosphere_of_the_Moon
      $endgroup$
      – PM 2Ring
      19 hours ago






    • 1




      $begingroup$
      From the list of FAQ for Stack Exchange sites one can see for example How do comment @replies work?
      $endgroup$
      – uhoh
      17 hours ago






    • 2




      $begingroup$
      Sorry, I didn't expect the discussion to get so long. But oddly, the "Would you like this to be moved to chat" thingy still hasn't appeared.
      $endgroup$
      – PM 2Ring
      10 hours ago


















    5












    $begingroup$

    update: This answer was written before the question was modified. I've tried to explain where a value like 10-17 Torr for deep space might come from, but it's since been dropped in lieu of 10-11 Torr at the Moon, which is probably a better way to formulate the question.



    I think the answer is the same, two points very far away can have very different pressures. They can coexist in the same solar system, just not right next to each other. I think "Why doesn't the Moon have at least a small atmosphere?" could also be an excellent, but very different question.





    In a comment the OP links to the presentation VACUUM (There’s nothing to it… ) written from the perspective of an engineer in the semiconductor manufacturing industry.



    Slide 6 gives examples of vacuum levels in different situations:



    Going down:




    • Low vacuum: 760 Torr to 1 x 10-3 Torr


      • Vacuum cleaner: to 600 Torr

      • Thermos bottle 10-3 Torr



    • High vacuum: 10-3 to 10-9 Torr


      • Electron microscope

      • Ion Implanter
        – Evaporator
        – Sputterer



    • Ultra high vacuum: 10-9 to 10-12 Torr


      • CERN LHC: 1 x 10-10 Torr

      • Moon’s surface: 1 x 10-11 Torr

      • Deep Space 1 x 10-17 Torr = 0.000,000,000,000,000,01 Torr




    So we can see that the value of 1 x 10-17 Torr is associated with a place in "Deep Space" which is (probably) beyond that of the Moon.



    Let's see if we can figure out where the author is getting that number.



    According to the Wikipedia article on the interstellar medium (space between stars, far away from solar systems and other things):




    In all phases, the interstellar medium is extremely tenuous by terrestrial standards. In cool, dense regions of the ISM, matter is primarily in molecular form, and reaches number densities of 106 molecules per cm3 (1 million molecules per cm3). In hot, diffuse regions of the ISM, matter is primarily ionized, and the density may be as low as 10−4 ions per cm3. Compare this with a number density of roughly 1019 molecules per cm3 for air at sea level, and 1010 molecules per cm3 (10 billion molecules per cm3) for a laboratory high-vacuum chamber.




    It is harder to talk about pressure than density because pressure is related to both number density and temperature. The atmosphere is more than 10x hotter than interstellar medium, so let's look for a number density ratio of 1/10 of 1000 Torr versus 10-17 Torr, or a ratio of 1019.



    If (according to Wikipedia) Earth's atmosphere has a density of 1019 per cm3, we're looking for a density of 1 per cm3. Checking Wikipedia we can see that there are components of the interstellar medium with number densities between 106 and 10-4.



    It looks like the value in the presentation a rough ballpark estimate, but isn't off by more than a handful of orders of magnitude ;-)




    How can such a vacuum coexist with the open system of earth’s atmosphere whereby debris from space can enter in?




    While these two pressures can coexist in the same universe, they don't coexist in proximity at all. The interstellar medium is very, very far away from Earth's atmosphere, on the order of a lightyear.



    Gravity keeps Earth's atmosphere nearby Earth, the solar system has gas produced by (and attracted by) the Sun's gravity. In interstellar space, there just isn't any source of gas, and what might have been there at one time has moved away, towards sources of gravity, over billions of years.






    share|improve this answer











    $endgroup$









    • 1




      $begingroup$
      @Autodidact You may find several questions and answers like that in those sites already, best to read them before asking something that's been answered. Here is some related math, but there are probably better places to find a useful and head-wrappable explanation. en.wikipedia.org/wiki/Scale_height
      $endgroup$
      – uhoh
      20 hours ago








    • 1




      $begingroup$
      Thank you for the link. If this question is not asked in the correct stack, my sincerest apologies. Also my edit was not intended as a moving target, I didn’t want it dismissed. As for the atmosphere pressure, while the link provides a distance it doesn’t explain my inquiry, what is it pressing upon to generate the distance? That is a question to which I cannot seem to find an answer to. Gravity seems to be the answer but it’s powers are conflicting as I’ve pointed out. It’s sufficiently strong to resist a vacuum of 10^-11 but weak enough to let gas particles up 8.5 km up into the atmosphere.
      $endgroup$
      – Autodidact
      19 hours ago






    • 1




      $begingroup$
      The Moon kind of does have an atmosphere, but it's very thin, and its total mass is under 10 tonnes. See en.wikipedia.org/wiki/Atmosphere_of_the_Moon
      $endgroup$
      – PM 2Ring
      19 hours ago






    • 1




      $begingroup$
      From the list of FAQ for Stack Exchange sites one can see for example How do comment @replies work?
      $endgroup$
      – uhoh
      17 hours ago






    • 2




      $begingroup$
      Sorry, I didn't expect the discussion to get so long. But oddly, the "Would you like this to be moved to chat" thingy still hasn't appeared.
      $endgroup$
      – PM 2Ring
      10 hours ago
















    5












    5








    5





    $begingroup$

    update: This answer was written before the question was modified. I've tried to explain where a value like 10-17 Torr for deep space might come from, but it's since been dropped in lieu of 10-11 Torr at the Moon, which is probably a better way to formulate the question.



    I think the answer is the same, two points very far away can have very different pressures. They can coexist in the same solar system, just not right next to each other. I think "Why doesn't the Moon have at least a small atmosphere?" could also be an excellent, but very different question.





    In a comment the OP links to the presentation VACUUM (There’s nothing to it… ) written from the perspective of an engineer in the semiconductor manufacturing industry.



    Slide 6 gives examples of vacuum levels in different situations:



    Going down:




    • Low vacuum: 760 Torr to 1 x 10-3 Torr


      • Vacuum cleaner: to 600 Torr

      • Thermos bottle 10-3 Torr



    • High vacuum: 10-3 to 10-9 Torr


      • Electron microscope

      • Ion Implanter
        – Evaporator
        – Sputterer



    • Ultra high vacuum: 10-9 to 10-12 Torr


      • CERN LHC: 1 x 10-10 Torr

      • Moon’s surface: 1 x 10-11 Torr

      • Deep Space 1 x 10-17 Torr = 0.000,000,000,000,000,01 Torr




    So we can see that the value of 1 x 10-17 Torr is associated with a place in "Deep Space" which is (probably) beyond that of the Moon.



    Let's see if we can figure out where the author is getting that number.



    According to the Wikipedia article on the interstellar medium (space between stars, far away from solar systems and other things):




    In all phases, the interstellar medium is extremely tenuous by terrestrial standards. In cool, dense regions of the ISM, matter is primarily in molecular form, and reaches number densities of 106 molecules per cm3 (1 million molecules per cm3). In hot, diffuse regions of the ISM, matter is primarily ionized, and the density may be as low as 10−4 ions per cm3. Compare this with a number density of roughly 1019 molecules per cm3 for air at sea level, and 1010 molecules per cm3 (10 billion molecules per cm3) for a laboratory high-vacuum chamber.




    It is harder to talk about pressure than density because pressure is related to both number density and temperature. The atmosphere is more than 10x hotter than interstellar medium, so let's look for a number density ratio of 1/10 of 1000 Torr versus 10-17 Torr, or a ratio of 1019.



    If (according to Wikipedia) Earth's atmosphere has a density of 1019 per cm3, we're looking for a density of 1 per cm3. Checking Wikipedia we can see that there are components of the interstellar medium with number densities between 106 and 10-4.



    It looks like the value in the presentation a rough ballpark estimate, but isn't off by more than a handful of orders of magnitude ;-)




    How can such a vacuum coexist with the open system of earth’s atmosphere whereby debris from space can enter in?




    While these two pressures can coexist in the same universe, they don't coexist in proximity at all. The interstellar medium is very, very far away from Earth's atmosphere, on the order of a lightyear.



    Gravity keeps Earth's atmosphere nearby Earth, the solar system has gas produced by (and attracted by) the Sun's gravity. In interstellar space, there just isn't any source of gas, and what might have been there at one time has moved away, towards sources of gravity, over billions of years.






    share|improve this answer











    $endgroup$



    update: This answer was written before the question was modified. I've tried to explain where a value like 10-17 Torr for deep space might come from, but it's since been dropped in lieu of 10-11 Torr at the Moon, which is probably a better way to formulate the question.



    I think the answer is the same, two points very far away can have very different pressures. They can coexist in the same solar system, just not right next to each other. I think "Why doesn't the Moon have at least a small atmosphere?" could also be an excellent, but very different question.





    In a comment the OP links to the presentation VACUUM (There’s nothing to it… ) written from the perspective of an engineer in the semiconductor manufacturing industry.



    Slide 6 gives examples of vacuum levels in different situations:



    Going down:




    • Low vacuum: 760 Torr to 1 x 10-3 Torr


      • Vacuum cleaner: to 600 Torr

      • Thermos bottle 10-3 Torr



    • High vacuum: 10-3 to 10-9 Torr


      • Electron microscope

      • Ion Implanter
        – Evaporator
        – Sputterer



    • Ultra high vacuum: 10-9 to 10-12 Torr


      • CERN LHC: 1 x 10-10 Torr

      • Moon’s surface: 1 x 10-11 Torr

      • Deep Space 1 x 10-17 Torr = 0.000,000,000,000,000,01 Torr




    So we can see that the value of 1 x 10-17 Torr is associated with a place in "Deep Space" which is (probably) beyond that of the Moon.



    Let's see if we can figure out where the author is getting that number.



    According to the Wikipedia article on the interstellar medium (space between stars, far away from solar systems and other things):




    In all phases, the interstellar medium is extremely tenuous by terrestrial standards. In cool, dense regions of the ISM, matter is primarily in molecular form, and reaches number densities of 106 molecules per cm3 (1 million molecules per cm3). In hot, diffuse regions of the ISM, matter is primarily ionized, and the density may be as low as 10−4 ions per cm3. Compare this with a number density of roughly 1019 molecules per cm3 for air at sea level, and 1010 molecules per cm3 (10 billion molecules per cm3) for a laboratory high-vacuum chamber.




    It is harder to talk about pressure than density because pressure is related to both number density and temperature. The atmosphere is more than 10x hotter than interstellar medium, so let's look for a number density ratio of 1/10 of 1000 Torr versus 10-17 Torr, or a ratio of 1019.



    If (according to Wikipedia) Earth's atmosphere has a density of 1019 per cm3, we're looking for a density of 1 per cm3. Checking Wikipedia we can see that there are components of the interstellar medium with number densities between 106 and 10-4.



    It looks like the value in the presentation a rough ballpark estimate, but isn't off by more than a handful of orders of magnitude ;-)




    How can such a vacuum coexist with the open system of earth’s atmosphere whereby debris from space can enter in?




    While these two pressures can coexist in the same universe, they don't coexist in proximity at all. The interstellar medium is very, very far away from Earth's atmosphere, on the order of a lightyear.



    Gravity keeps Earth's atmosphere nearby Earth, the solar system has gas produced by (and attracted by) the Sun's gravity. In interstellar space, there just isn't any source of gas, and what might have been there at one time has moved away, towards sources of gravity, over billions of years.







    share|improve this answer














    share|improve this answer



    share|improve this answer








    edited 11 hours ago









    Peter A. Schneider

    49939




    49939










    answered 20 hours ago









    uhohuhoh

    6,05721761




    6,05721761








    • 1




      $begingroup$
      @Autodidact You may find several questions and answers like that in those sites already, best to read them before asking something that's been answered. Here is some related math, but there are probably better places to find a useful and head-wrappable explanation. en.wikipedia.org/wiki/Scale_height
      $endgroup$
      – uhoh
      20 hours ago








    • 1




      $begingroup$
      Thank you for the link. If this question is not asked in the correct stack, my sincerest apologies. Also my edit was not intended as a moving target, I didn’t want it dismissed. As for the atmosphere pressure, while the link provides a distance it doesn’t explain my inquiry, what is it pressing upon to generate the distance? That is a question to which I cannot seem to find an answer to. Gravity seems to be the answer but it’s powers are conflicting as I’ve pointed out. It’s sufficiently strong to resist a vacuum of 10^-11 but weak enough to let gas particles up 8.5 km up into the atmosphere.
      $endgroup$
      – Autodidact
      19 hours ago






    • 1




      $begingroup$
      The Moon kind of does have an atmosphere, but it's very thin, and its total mass is under 10 tonnes. See en.wikipedia.org/wiki/Atmosphere_of_the_Moon
      $endgroup$
      – PM 2Ring
      19 hours ago






    • 1




      $begingroup$
      From the list of FAQ for Stack Exchange sites one can see for example How do comment @replies work?
      $endgroup$
      – uhoh
      17 hours ago






    • 2




      $begingroup$
      Sorry, I didn't expect the discussion to get so long. But oddly, the "Would you like this to be moved to chat" thingy still hasn't appeared.
      $endgroup$
      – PM 2Ring
      10 hours ago
















    • 1




      $begingroup$
      @Autodidact You may find several questions and answers like that in those sites already, best to read them before asking something that's been answered. Here is some related math, but there are probably better places to find a useful and head-wrappable explanation. en.wikipedia.org/wiki/Scale_height
      $endgroup$
      – uhoh
      20 hours ago








    • 1




      $begingroup$
      Thank you for the link. If this question is not asked in the correct stack, my sincerest apologies. Also my edit was not intended as a moving target, I didn’t want it dismissed. As for the atmosphere pressure, while the link provides a distance it doesn’t explain my inquiry, what is it pressing upon to generate the distance? That is a question to which I cannot seem to find an answer to. Gravity seems to be the answer but it’s powers are conflicting as I’ve pointed out. It’s sufficiently strong to resist a vacuum of 10^-11 but weak enough to let gas particles up 8.5 km up into the atmosphere.
      $endgroup$
      – Autodidact
      19 hours ago






    • 1




      $begingroup$
      The Moon kind of does have an atmosphere, but it's very thin, and its total mass is under 10 tonnes. See en.wikipedia.org/wiki/Atmosphere_of_the_Moon
      $endgroup$
      – PM 2Ring
      19 hours ago






    • 1




      $begingroup$
      From the list of FAQ for Stack Exchange sites one can see for example How do comment @replies work?
      $endgroup$
      – uhoh
      17 hours ago






    • 2




      $begingroup$
      Sorry, I didn't expect the discussion to get so long. But oddly, the "Would you like this to be moved to chat" thingy still hasn't appeared.
      $endgroup$
      – PM 2Ring
      10 hours ago










    1




    1




    $begingroup$
    @Autodidact You may find several questions and answers like that in those sites already, best to read them before asking something that's been answered. Here is some related math, but there are probably better places to find a useful and head-wrappable explanation. en.wikipedia.org/wiki/Scale_height
    $endgroup$
    – uhoh
    20 hours ago






    $begingroup$
    @Autodidact You may find several questions and answers like that in those sites already, best to read them before asking something that's been answered. Here is some related math, but there are probably better places to find a useful and head-wrappable explanation. en.wikipedia.org/wiki/Scale_height
    $endgroup$
    – uhoh
    20 hours ago






    1




    1




    $begingroup$
    Thank you for the link. If this question is not asked in the correct stack, my sincerest apologies. Also my edit was not intended as a moving target, I didn’t want it dismissed. As for the atmosphere pressure, while the link provides a distance it doesn’t explain my inquiry, what is it pressing upon to generate the distance? That is a question to which I cannot seem to find an answer to. Gravity seems to be the answer but it’s powers are conflicting as I’ve pointed out. It’s sufficiently strong to resist a vacuum of 10^-11 but weak enough to let gas particles up 8.5 km up into the atmosphere.
    $endgroup$
    – Autodidact
    19 hours ago




    $begingroup$
    Thank you for the link. If this question is not asked in the correct stack, my sincerest apologies. Also my edit was not intended as a moving target, I didn’t want it dismissed. As for the atmosphere pressure, while the link provides a distance it doesn’t explain my inquiry, what is it pressing upon to generate the distance? That is a question to which I cannot seem to find an answer to. Gravity seems to be the answer but it’s powers are conflicting as I’ve pointed out. It’s sufficiently strong to resist a vacuum of 10^-11 but weak enough to let gas particles up 8.5 km up into the atmosphere.
    $endgroup$
    – Autodidact
    19 hours ago




    1




    1




    $begingroup$
    The Moon kind of does have an atmosphere, but it's very thin, and its total mass is under 10 tonnes. See en.wikipedia.org/wiki/Atmosphere_of_the_Moon
    $endgroup$
    – PM 2Ring
    19 hours ago




    $begingroup$
    The Moon kind of does have an atmosphere, but it's very thin, and its total mass is under 10 tonnes. See en.wikipedia.org/wiki/Atmosphere_of_the_Moon
    $endgroup$
    – PM 2Ring
    19 hours ago




    1




    1




    $begingroup$
    From the list of FAQ for Stack Exchange sites one can see for example How do comment @replies work?
    $endgroup$
    – uhoh
    17 hours ago




    $begingroup$
    From the list of FAQ for Stack Exchange sites one can see for example How do comment @replies work?
    $endgroup$
    – uhoh
    17 hours ago




    2




    2




    $begingroup$
    Sorry, I didn't expect the discussion to get so long. But oddly, the "Would you like this to be moved to chat" thingy still hasn't appeared.
    $endgroup$
    – PM 2Ring
    10 hours ago






    $begingroup$
    Sorry, I didn't expect the discussion to get so long. But oddly, the "Would you like this to be moved to chat" thingy still hasn't appeared.
    $endgroup$
    – PM 2Ring
    10 hours ago













    4












    $begingroup$

    The underlying reason that the molecules of Earth's atmosphere do not fly away into the surrounding vacuum is that they are slower than the escape velocity, which would be 11200 m/s. The typical molecule speed at ground level and room temperature appears to be 500 m/s. If it had a free path such a molecule could fly vertically for ~50s before starting to fall back, with an average velocity of 250 m/s, thus reaching an altitude of 12 or 13 km. (In reality it would collide with other molecules on the way, transferring some kinetic energy to them, so that they could in turn rise higher. Obviously, molecules at the outer fringes of the atmosphere are the real escape candidates.)



    Molecules which are fast enough surely do escape Earth's gravity well. Some may have been accelerated by particles of the solar wind, some may just have been on the long tail of the standard distribution. The latter is more likely for light atoms and molecules which are faster, like Helium and Hydrogen. Hydrogen's average speed at room temperature is perhaps 2000 m/s. These gases have indeed mostly left Earth for good long ago.



    (By the way, the solar wind would likely "blow away" our atmosphere in the long run — as it did on Mars — if it weren't deflected by the Earth's magnetic field.)






    share|improve this answer











    $endgroup$


















      4












      $begingroup$

      The underlying reason that the molecules of Earth's atmosphere do not fly away into the surrounding vacuum is that they are slower than the escape velocity, which would be 11200 m/s. The typical molecule speed at ground level and room temperature appears to be 500 m/s. If it had a free path such a molecule could fly vertically for ~50s before starting to fall back, with an average velocity of 250 m/s, thus reaching an altitude of 12 or 13 km. (In reality it would collide with other molecules on the way, transferring some kinetic energy to them, so that they could in turn rise higher. Obviously, molecules at the outer fringes of the atmosphere are the real escape candidates.)



      Molecules which are fast enough surely do escape Earth's gravity well. Some may have been accelerated by particles of the solar wind, some may just have been on the long tail of the standard distribution. The latter is more likely for light atoms and molecules which are faster, like Helium and Hydrogen. Hydrogen's average speed at room temperature is perhaps 2000 m/s. These gases have indeed mostly left Earth for good long ago.



      (By the way, the solar wind would likely "blow away" our atmosphere in the long run — as it did on Mars — if it weren't deflected by the Earth's magnetic field.)






      share|improve this answer











      $endgroup$
















        4












        4








        4





        $begingroup$

        The underlying reason that the molecules of Earth's atmosphere do not fly away into the surrounding vacuum is that they are slower than the escape velocity, which would be 11200 m/s. The typical molecule speed at ground level and room temperature appears to be 500 m/s. If it had a free path such a molecule could fly vertically for ~50s before starting to fall back, with an average velocity of 250 m/s, thus reaching an altitude of 12 or 13 km. (In reality it would collide with other molecules on the way, transferring some kinetic energy to them, so that they could in turn rise higher. Obviously, molecules at the outer fringes of the atmosphere are the real escape candidates.)



        Molecules which are fast enough surely do escape Earth's gravity well. Some may have been accelerated by particles of the solar wind, some may just have been on the long tail of the standard distribution. The latter is more likely for light atoms and molecules which are faster, like Helium and Hydrogen. Hydrogen's average speed at room temperature is perhaps 2000 m/s. These gases have indeed mostly left Earth for good long ago.



        (By the way, the solar wind would likely "blow away" our atmosphere in the long run — as it did on Mars — if it weren't deflected by the Earth's magnetic field.)






        share|improve this answer











        $endgroup$



        The underlying reason that the molecules of Earth's atmosphere do not fly away into the surrounding vacuum is that they are slower than the escape velocity, which would be 11200 m/s. The typical molecule speed at ground level and room temperature appears to be 500 m/s. If it had a free path such a molecule could fly vertically for ~50s before starting to fall back, with an average velocity of 250 m/s, thus reaching an altitude of 12 or 13 km. (In reality it would collide with other molecules on the way, transferring some kinetic energy to them, so that they could in turn rise higher. Obviously, molecules at the outer fringes of the atmosphere are the real escape candidates.)



        Molecules which are fast enough surely do escape Earth's gravity well. Some may have been accelerated by particles of the solar wind, some may just have been on the long tail of the standard distribution. The latter is more likely for light atoms and molecules which are faster, like Helium and Hydrogen. Hydrogen's average speed at room temperature is perhaps 2000 m/s. These gases have indeed mostly left Earth for good long ago.



        (By the way, the solar wind would likely "blow away" our atmosphere in the long run — as it did on Mars — if it weren't deflected by the Earth's magnetic field.)







        share|improve this answer














        share|improve this answer



        share|improve this answer








        edited 11 hours ago

























        answered 11 hours ago









        Peter A. SchneiderPeter A. Schneider

        49939




        49939























            1












            $begingroup$

            I think uhoh covered proximity but just to induce equilibrium to further elaborate:

            First of all, positive pressure and negative pressure are just terminology based on where we started i.e. 1 atm and above/below, we just went along. There is zero pressure and gradually moving up from that. Say, perfect vacuum starts at zero pressure and move up as comes across high pressure open system. Things are always at equilibrium and if you want to move something from low pressure to high, you do some work as you have mentioned second law of thermodynamics. Gravity does that work in this case till some point. Ignore everything, lets say there is one good looking blue dot aka earth, and as you move close, gravity starts to get stronger. So, it will invite more molecule to be cuddly and at same time pressure gradient will move gas the other way. Eventually they will reach at equilibrium i.e. same transference. This is true for any height from earth. In absence of such equilibrium earth would loose its atmosphere or gain more (may be case at astronomical time scale). We start at one (gravity rules) to continuously (important) merge to space vacuum with equilibrium all along the line. Once gas truly escape earth gravity (i.e. temperature movement is way stronger than earth gravity), it has no reason to hang around. Same way we can gain by unsuspecting wandering gas molecules. Earth atmosphere looks at equilibrium at human timescale. But earth loose and gain as these forces continuously change with distance.






            share|improve this answer








            New contributor




            HR04375439 is a new contributor to this site. Take care in asking for clarification, commenting, and answering.
            Check out our Code of Conduct.






            $endgroup$













            • $begingroup$
              At this point I really neeed citations to decide on the best answer. Could you cite your response or are you merely theorizing?
              $endgroup$
              – Autodidact
              6 hours ago
















            1












            $begingroup$

            I think uhoh covered proximity but just to induce equilibrium to further elaborate:

            First of all, positive pressure and negative pressure are just terminology based on where we started i.e. 1 atm and above/below, we just went along. There is zero pressure and gradually moving up from that. Say, perfect vacuum starts at zero pressure and move up as comes across high pressure open system. Things are always at equilibrium and if you want to move something from low pressure to high, you do some work as you have mentioned second law of thermodynamics. Gravity does that work in this case till some point. Ignore everything, lets say there is one good looking blue dot aka earth, and as you move close, gravity starts to get stronger. So, it will invite more molecule to be cuddly and at same time pressure gradient will move gas the other way. Eventually they will reach at equilibrium i.e. same transference. This is true for any height from earth. In absence of such equilibrium earth would loose its atmosphere or gain more (may be case at astronomical time scale). We start at one (gravity rules) to continuously (important) merge to space vacuum with equilibrium all along the line. Once gas truly escape earth gravity (i.e. temperature movement is way stronger than earth gravity), it has no reason to hang around. Same way we can gain by unsuspecting wandering gas molecules. Earth atmosphere looks at equilibrium at human timescale. But earth loose and gain as these forces continuously change with distance.






            share|improve this answer








            New contributor




            HR04375439 is a new contributor to this site. Take care in asking for clarification, commenting, and answering.
            Check out our Code of Conduct.






            $endgroup$













            • $begingroup$
              At this point I really neeed citations to decide on the best answer. Could you cite your response or are you merely theorizing?
              $endgroup$
              – Autodidact
              6 hours ago














            1












            1








            1





            $begingroup$

            I think uhoh covered proximity but just to induce equilibrium to further elaborate:

            First of all, positive pressure and negative pressure are just terminology based on where we started i.e. 1 atm and above/below, we just went along. There is zero pressure and gradually moving up from that. Say, perfect vacuum starts at zero pressure and move up as comes across high pressure open system. Things are always at equilibrium and if you want to move something from low pressure to high, you do some work as you have mentioned second law of thermodynamics. Gravity does that work in this case till some point. Ignore everything, lets say there is one good looking blue dot aka earth, and as you move close, gravity starts to get stronger. So, it will invite more molecule to be cuddly and at same time pressure gradient will move gas the other way. Eventually they will reach at equilibrium i.e. same transference. This is true for any height from earth. In absence of such equilibrium earth would loose its atmosphere or gain more (may be case at astronomical time scale). We start at one (gravity rules) to continuously (important) merge to space vacuum with equilibrium all along the line. Once gas truly escape earth gravity (i.e. temperature movement is way stronger than earth gravity), it has no reason to hang around. Same way we can gain by unsuspecting wandering gas molecules. Earth atmosphere looks at equilibrium at human timescale. But earth loose and gain as these forces continuously change with distance.






            share|improve this answer








            New contributor




            HR04375439 is a new contributor to this site. Take care in asking for clarification, commenting, and answering.
            Check out our Code of Conduct.






            $endgroup$



            I think uhoh covered proximity but just to induce equilibrium to further elaborate:

            First of all, positive pressure and negative pressure are just terminology based on where we started i.e. 1 atm and above/below, we just went along. There is zero pressure and gradually moving up from that. Say, perfect vacuum starts at zero pressure and move up as comes across high pressure open system. Things are always at equilibrium and if you want to move something from low pressure to high, you do some work as you have mentioned second law of thermodynamics. Gravity does that work in this case till some point. Ignore everything, lets say there is one good looking blue dot aka earth, and as you move close, gravity starts to get stronger. So, it will invite more molecule to be cuddly and at same time pressure gradient will move gas the other way. Eventually they will reach at equilibrium i.e. same transference. This is true for any height from earth. In absence of such equilibrium earth would loose its atmosphere or gain more (may be case at astronomical time scale). We start at one (gravity rules) to continuously (important) merge to space vacuum with equilibrium all along the line. Once gas truly escape earth gravity (i.e. temperature movement is way stronger than earth gravity), it has no reason to hang around. Same way we can gain by unsuspecting wandering gas molecules. Earth atmosphere looks at equilibrium at human timescale. But earth loose and gain as these forces continuously change with distance.







            share|improve this answer








            New contributor




            HR04375439 is a new contributor to this site. Take care in asking for clarification, commenting, and answering.
            Check out our Code of Conduct.









            share|improve this answer



            share|improve this answer






            New contributor




            HR04375439 is a new contributor to this site. Take care in asking for clarification, commenting, and answering.
            Check out our Code of Conduct.









            answered 11 hours ago









            HR04375439HR04375439

            313




            313




            New contributor




            HR04375439 is a new contributor to this site. Take care in asking for clarification, commenting, and answering.
            Check out our Code of Conduct.





            New contributor





            HR04375439 is a new contributor to this site. Take care in asking for clarification, commenting, and answering.
            Check out our Code of Conduct.






            HR04375439 is a new contributor to this site. Take care in asking for clarification, commenting, and answering.
            Check out our Code of Conduct.












            • $begingroup$
              At this point I really neeed citations to decide on the best answer. Could you cite your response or are you merely theorizing?
              $endgroup$
              – Autodidact
              6 hours ago


















            • $begingroup$
              At this point I really neeed citations to decide on the best answer. Could you cite your response or are you merely theorizing?
              $endgroup$
              – Autodidact
              6 hours ago
















            $begingroup$
            At this point I really neeed citations to decide on the best answer. Could you cite your response or are you merely theorizing?
            $endgroup$
            – Autodidact
            6 hours ago




            $begingroup$
            At this point I really neeed citations to decide on the best answer. Could you cite your response or are you merely theorizing?
            $endgroup$
            – Autodidact
            6 hours ago











            0












            $begingroup$

            Your assertion that our atmosphere doesn't escape is wrong.



            Helium and Hydrogen atoms have a low enough mass that they do have an escape velocity at the temperatures on the edge of our atmosphere. This means that when those gasses are released, if they fail to react on their way out, then they will be lost forever to the planet. This is why when you look at our atmosphere we just don't have any.



            You also seem to assert that there's a "line" where it's the high pressure of our atmosphere on one side, and a low pressure of space on the other; that line doesn't exist. The pressure is a gradient, in the same way as you swim down in a pool, at the top the pressure is low; at the bottom it's high; and it changes steadily as you traverse between the two. Hence why climbers of Everest need to carry oxygen.



            The heavier gasses don't escape for the same reason that a rock you throw doesn't escape. It requires energy to escape the gravitational well of Earth; and they don't have it; currently. Of course, as the atmosphere heats up from global warming, heavier and heavier particles will gain the energy to leave our atmosphere...






            share|improve this answer








            New contributor




            UKMonkey is a new contributor to this site. Take care in asking for clarification, commenting, and answering.
            Check out our Code of Conduct.






            $endgroup$













            • $begingroup$
              Ok, please cite research papers on the gradient you speak of. Also please cite how the second law of thermodynamics is suspended because of insufficient energy especially when there is no “line”. And also how pressure is maintained without a container in the open system of earth’s atmosphere. Cite whatever you think is correct but please cite it.
              $endgroup$
              – Autodidact
              5 hours ago












            • $begingroup$
              @Autodidact Why do you need citations for the pressure gradients? Surely you can take it as read? And the pressure is maintained "in the open system of the Earth's atmosphere" by gravity, as already explained. You could search for "baratropic law" or "atmosphere scale height" for more information on that.
              $endgroup$
              – Jim421616
              4 hours ago












            • $begingroup$
              Because he made an assertion and I want it backed up with science. “You also seem to assert that there's a "line" where it's the high pressure of our atmosphere on one side, and a low pressure of space on the other; that line doesn't exist. The pressure is a gradient“. That’s why I require it. Secondly I didn’t claim a line, I’m asking how this high pressure doesn’t adhere to the second law of thermodynamics, for such a gradient to even be possible in the first place. Because I’m getting gravity, mixed with buoyancy, density, mixed with pressure, gradients and stratification. Which is it?
              $endgroup$
              – Autodidact
              4 hours ago












            • $begingroup$
              @Autodidact I wouldn't start with reading papers. I get the strong impression that your understanding of the basics isn't sufficient to go through a paper and make sense of it. I would suggest just reading en.wikipedia.org/wiki/Atmospheric_pressure Consider it this way. At sea level, you have all the atmosphere above you pushing down. There's a lot of air above you, it can push down really quite hard. If you go up 500m, you have that much less air being pulled down by gravity onto you; and thus the pressure drops.
              $endgroup$
              – UKMonkey
              2 hours ago










            • $begingroup$
              @Autodidact Your comment about "buoyancy" suggests to me you're watching flat earth videos. They have a very very poor understanding of physics; and are often intentionally misleading. You may find it better to take a balloon to a swimming pool, fill it with air, and then measure the size of it at different depths to demonstrate the pressure gradient.
              $endgroup$
              – UKMonkey
              2 hours ago
















            0












            $begingroup$

            Your assertion that our atmosphere doesn't escape is wrong.



            Helium and Hydrogen atoms have a low enough mass that they do have an escape velocity at the temperatures on the edge of our atmosphere. This means that when those gasses are released, if they fail to react on their way out, then they will be lost forever to the planet. This is why when you look at our atmosphere we just don't have any.



            You also seem to assert that there's a "line" where it's the high pressure of our atmosphere on one side, and a low pressure of space on the other; that line doesn't exist. The pressure is a gradient, in the same way as you swim down in a pool, at the top the pressure is low; at the bottom it's high; and it changes steadily as you traverse between the two. Hence why climbers of Everest need to carry oxygen.



            The heavier gasses don't escape for the same reason that a rock you throw doesn't escape. It requires energy to escape the gravitational well of Earth; and they don't have it; currently. Of course, as the atmosphere heats up from global warming, heavier and heavier particles will gain the energy to leave our atmosphere...






            share|improve this answer








            New contributor




            UKMonkey is a new contributor to this site. Take care in asking for clarification, commenting, and answering.
            Check out our Code of Conduct.






            $endgroup$













            • $begingroup$
              Ok, please cite research papers on the gradient you speak of. Also please cite how the second law of thermodynamics is suspended because of insufficient energy especially when there is no “line”. And also how pressure is maintained without a container in the open system of earth’s atmosphere. Cite whatever you think is correct but please cite it.
              $endgroup$
              – Autodidact
              5 hours ago












            • $begingroup$
              @Autodidact Why do you need citations for the pressure gradients? Surely you can take it as read? And the pressure is maintained "in the open system of the Earth's atmosphere" by gravity, as already explained. You could search for "baratropic law" or "atmosphere scale height" for more information on that.
              $endgroup$
              – Jim421616
              4 hours ago












            • $begingroup$
              Because he made an assertion and I want it backed up with science. “You also seem to assert that there's a "line" where it's the high pressure of our atmosphere on one side, and a low pressure of space on the other; that line doesn't exist. The pressure is a gradient“. That’s why I require it. Secondly I didn’t claim a line, I’m asking how this high pressure doesn’t adhere to the second law of thermodynamics, for such a gradient to even be possible in the first place. Because I’m getting gravity, mixed with buoyancy, density, mixed with pressure, gradients and stratification. Which is it?
              $endgroup$
              – Autodidact
              4 hours ago












            • $begingroup$
              @Autodidact I wouldn't start with reading papers. I get the strong impression that your understanding of the basics isn't sufficient to go through a paper and make sense of it. I would suggest just reading en.wikipedia.org/wiki/Atmospheric_pressure Consider it this way. At sea level, you have all the atmosphere above you pushing down. There's a lot of air above you, it can push down really quite hard. If you go up 500m, you have that much less air being pulled down by gravity onto you; and thus the pressure drops.
              $endgroup$
              – UKMonkey
              2 hours ago










            • $begingroup$
              @Autodidact Your comment about "buoyancy" suggests to me you're watching flat earth videos. They have a very very poor understanding of physics; and are often intentionally misleading. You may find it better to take a balloon to a swimming pool, fill it with air, and then measure the size of it at different depths to demonstrate the pressure gradient.
              $endgroup$
              – UKMonkey
              2 hours ago














            0












            0








            0





            $begingroup$

            Your assertion that our atmosphere doesn't escape is wrong.



            Helium and Hydrogen atoms have a low enough mass that they do have an escape velocity at the temperatures on the edge of our atmosphere. This means that when those gasses are released, if they fail to react on their way out, then they will be lost forever to the planet. This is why when you look at our atmosphere we just don't have any.



            You also seem to assert that there's a "line" where it's the high pressure of our atmosphere on one side, and a low pressure of space on the other; that line doesn't exist. The pressure is a gradient, in the same way as you swim down in a pool, at the top the pressure is low; at the bottom it's high; and it changes steadily as you traverse between the two. Hence why climbers of Everest need to carry oxygen.



            The heavier gasses don't escape for the same reason that a rock you throw doesn't escape. It requires energy to escape the gravitational well of Earth; and they don't have it; currently. Of course, as the atmosphere heats up from global warming, heavier and heavier particles will gain the energy to leave our atmosphere...






            share|improve this answer








            New contributor




            UKMonkey is a new contributor to this site. Take care in asking for clarification, commenting, and answering.
            Check out our Code of Conduct.






            $endgroup$



            Your assertion that our atmosphere doesn't escape is wrong.



            Helium and Hydrogen atoms have a low enough mass that they do have an escape velocity at the temperatures on the edge of our atmosphere. This means that when those gasses are released, if they fail to react on their way out, then they will be lost forever to the planet. This is why when you look at our atmosphere we just don't have any.



            You also seem to assert that there's a "line" where it's the high pressure of our atmosphere on one side, and a low pressure of space on the other; that line doesn't exist. The pressure is a gradient, in the same way as you swim down in a pool, at the top the pressure is low; at the bottom it's high; and it changes steadily as you traverse between the two. Hence why climbers of Everest need to carry oxygen.



            The heavier gasses don't escape for the same reason that a rock you throw doesn't escape. It requires energy to escape the gravitational well of Earth; and they don't have it; currently. Of course, as the atmosphere heats up from global warming, heavier and heavier particles will gain the energy to leave our atmosphere...







            share|improve this answer








            New contributor




            UKMonkey is a new contributor to this site. Take care in asking for clarification, commenting, and answering.
            Check out our Code of Conduct.









            share|improve this answer



            share|improve this answer






            New contributor




            UKMonkey is a new contributor to this site. Take care in asking for clarification, commenting, and answering.
            Check out our Code of Conduct.









            answered 5 hours ago









            UKMonkeyUKMonkey

            1012




            1012




            New contributor




            UKMonkey is a new contributor to this site. Take care in asking for clarification, commenting, and answering.
            Check out our Code of Conduct.





            New contributor





            UKMonkey is a new contributor to this site. Take care in asking for clarification, commenting, and answering.
            Check out our Code of Conduct.






            UKMonkey is a new contributor to this site. Take care in asking for clarification, commenting, and answering.
            Check out our Code of Conduct.












            • $begingroup$
              Ok, please cite research papers on the gradient you speak of. Also please cite how the second law of thermodynamics is suspended because of insufficient energy especially when there is no “line”. And also how pressure is maintained without a container in the open system of earth’s atmosphere. Cite whatever you think is correct but please cite it.
              $endgroup$
              – Autodidact
              5 hours ago












            • $begingroup$
              @Autodidact Why do you need citations for the pressure gradients? Surely you can take it as read? And the pressure is maintained "in the open system of the Earth's atmosphere" by gravity, as already explained. You could search for "baratropic law" or "atmosphere scale height" for more information on that.
              $endgroup$
              – Jim421616
              4 hours ago












            • $begingroup$
              Because he made an assertion and I want it backed up with science. “You also seem to assert that there's a "line" where it's the high pressure of our atmosphere on one side, and a low pressure of space on the other; that line doesn't exist. The pressure is a gradient“. That’s why I require it. Secondly I didn’t claim a line, I’m asking how this high pressure doesn’t adhere to the second law of thermodynamics, for such a gradient to even be possible in the first place. Because I’m getting gravity, mixed with buoyancy, density, mixed with pressure, gradients and stratification. Which is it?
              $endgroup$
              – Autodidact
              4 hours ago












            • $begingroup$
              @Autodidact I wouldn't start with reading papers. I get the strong impression that your understanding of the basics isn't sufficient to go through a paper and make sense of it. I would suggest just reading en.wikipedia.org/wiki/Atmospheric_pressure Consider it this way. At sea level, you have all the atmosphere above you pushing down. There's a lot of air above you, it can push down really quite hard. If you go up 500m, you have that much less air being pulled down by gravity onto you; and thus the pressure drops.
              $endgroup$
              – UKMonkey
              2 hours ago










            • $begingroup$
              @Autodidact Your comment about "buoyancy" suggests to me you're watching flat earth videos. They have a very very poor understanding of physics; and are often intentionally misleading. You may find it better to take a balloon to a swimming pool, fill it with air, and then measure the size of it at different depths to demonstrate the pressure gradient.
              $endgroup$
              – UKMonkey
              2 hours ago


















            • $begingroup$
              Ok, please cite research papers on the gradient you speak of. Also please cite how the second law of thermodynamics is suspended because of insufficient energy especially when there is no “line”. And also how pressure is maintained without a container in the open system of earth’s atmosphere. Cite whatever you think is correct but please cite it.
              $endgroup$
              – Autodidact
              5 hours ago












            • $begingroup$
              @Autodidact Why do you need citations for the pressure gradients? Surely you can take it as read? And the pressure is maintained "in the open system of the Earth's atmosphere" by gravity, as already explained. You could search for "baratropic law" or "atmosphere scale height" for more information on that.
              $endgroup$
              – Jim421616
              4 hours ago












            • $begingroup$
              Because he made an assertion and I want it backed up with science. “You also seem to assert that there's a "line" where it's the high pressure of our atmosphere on one side, and a low pressure of space on the other; that line doesn't exist. The pressure is a gradient“. That’s why I require it. Secondly I didn’t claim a line, I’m asking how this high pressure doesn’t adhere to the second law of thermodynamics, for such a gradient to even be possible in the first place. Because I’m getting gravity, mixed with buoyancy, density, mixed with pressure, gradients and stratification. Which is it?
              $endgroup$
              – Autodidact
              4 hours ago












            • $begingroup$
              @Autodidact I wouldn't start with reading papers. I get the strong impression that your understanding of the basics isn't sufficient to go through a paper and make sense of it. I would suggest just reading en.wikipedia.org/wiki/Atmospheric_pressure Consider it this way. At sea level, you have all the atmosphere above you pushing down. There's a lot of air above you, it can push down really quite hard. If you go up 500m, you have that much less air being pulled down by gravity onto you; and thus the pressure drops.
              $endgroup$
              – UKMonkey
              2 hours ago










            • $begingroup$
              @Autodidact Your comment about "buoyancy" suggests to me you're watching flat earth videos. They have a very very poor understanding of physics; and are often intentionally misleading. You may find it better to take a balloon to a swimming pool, fill it with air, and then measure the size of it at different depths to demonstrate the pressure gradient.
              $endgroup$
              – UKMonkey
              2 hours ago
















            $begingroup$
            Ok, please cite research papers on the gradient you speak of. Also please cite how the second law of thermodynamics is suspended because of insufficient energy especially when there is no “line”. And also how pressure is maintained without a container in the open system of earth’s atmosphere. Cite whatever you think is correct but please cite it.
            $endgroup$
            – Autodidact
            5 hours ago






            $begingroup$
            Ok, please cite research papers on the gradient you speak of. Also please cite how the second law of thermodynamics is suspended because of insufficient energy especially when there is no “line”. And also how pressure is maintained without a container in the open system of earth’s atmosphere. Cite whatever you think is correct but please cite it.
            $endgroup$
            – Autodidact
            5 hours ago














            $begingroup$
            @Autodidact Why do you need citations for the pressure gradients? Surely you can take it as read? And the pressure is maintained "in the open system of the Earth's atmosphere" by gravity, as already explained. You could search for "baratropic law" or "atmosphere scale height" for more information on that.
            $endgroup$
            – Jim421616
            4 hours ago






            $begingroup$
            @Autodidact Why do you need citations for the pressure gradients? Surely you can take it as read? And the pressure is maintained "in the open system of the Earth's atmosphere" by gravity, as already explained. You could search for "baratropic law" or "atmosphere scale height" for more information on that.
            $endgroup$
            – Jim421616
            4 hours ago














            $begingroup$
            Because he made an assertion and I want it backed up with science. “You also seem to assert that there's a "line" where it's the high pressure of our atmosphere on one side, and a low pressure of space on the other; that line doesn't exist. The pressure is a gradient“. That’s why I require it. Secondly I didn’t claim a line, I’m asking how this high pressure doesn’t adhere to the second law of thermodynamics, for such a gradient to even be possible in the first place. Because I’m getting gravity, mixed with buoyancy, density, mixed with pressure, gradients and stratification. Which is it?
            $endgroup$
            – Autodidact
            4 hours ago






            $begingroup$
            Because he made an assertion and I want it backed up with science. “You also seem to assert that there's a "line" where it's the high pressure of our atmosphere on one side, and a low pressure of space on the other; that line doesn't exist. The pressure is a gradient“. That’s why I require it. Secondly I didn’t claim a line, I’m asking how this high pressure doesn’t adhere to the second law of thermodynamics, for such a gradient to even be possible in the first place. Because I’m getting gravity, mixed with buoyancy, density, mixed with pressure, gradients and stratification. Which is it?
            $endgroup$
            – Autodidact
            4 hours ago














            $begingroup$
            @Autodidact I wouldn't start with reading papers. I get the strong impression that your understanding of the basics isn't sufficient to go through a paper and make sense of it. I would suggest just reading en.wikipedia.org/wiki/Atmospheric_pressure Consider it this way. At sea level, you have all the atmosphere above you pushing down. There's a lot of air above you, it can push down really quite hard. If you go up 500m, you have that much less air being pulled down by gravity onto you; and thus the pressure drops.
            $endgroup$
            – UKMonkey
            2 hours ago




            $begingroup$
            @Autodidact I wouldn't start with reading papers. I get the strong impression that your understanding of the basics isn't sufficient to go through a paper and make sense of it. I would suggest just reading en.wikipedia.org/wiki/Atmospheric_pressure Consider it this way. At sea level, you have all the atmosphere above you pushing down. There's a lot of air above you, it can push down really quite hard. If you go up 500m, you have that much less air being pulled down by gravity onto you; and thus the pressure drops.
            $endgroup$
            – UKMonkey
            2 hours ago












            $begingroup$
            @Autodidact Your comment about "buoyancy" suggests to me you're watching flat earth videos. They have a very very poor understanding of physics; and are often intentionally misleading. You may find it better to take a balloon to a swimming pool, fill it with air, and then measure the size of it at different depths to demonstrate the pressure gradient.
            $endgroup$
            – UKMonkey
            2 hours ago




            $begingroup$
            @Autodidact Your comment about "buoyancy" suggests to me you're watching flat earth videos. They have a very very poor understanding of physics; and are often intentionally misleading. You may find it better to take a balloon to a swimming pool, fill it with air, and then measure the size of it at different depths to demonstrate the pressure gradient.
            $endgroup$
            – UKMonkey
            2 hours ago










            Autodidact is a new contributor. Be nice, and check out our Code of Conduct.










            draft saved

            draft discarded


















            Autodidact is a new contributor. Be nice, and check out our Code of Conduct.













            Autodidact is a new contributor. Be nice, and check out our Code of Conduct.












            Autodidact is a new contributor. Be nice, and check out our Code of Conduct.
















            Thanks for contributing an answer to Astronomy Stack Exchange!


            • Please be sure to answer the question. Provide details and share your research!

            But avoid



            • Asking for help, clarification, or responding to other answers.

            • Making statements based on opinion; back them up with references or personal experience.


            Use MathJax to format equations. MathJax reference.


            To learn more, see our tips on writing great answers.




            draft saved


            draft discarded














            StackExchange.ready(
            function () {
            StackExchange.openid.initPostLogin('.new-post-login', 'https%3a%2f%2fastronomy.stackexchange.com%2fquestions%2f29832%2fvacuum-of-space%23new-answer', 'question_page');
            }
            );

            Post as a guest















            Required, but never shown





















































            Required, but never shown














            Required, but never shown












            Required, but never shown







            Required, but never shown

































            Required, but never shown














            Required, but never shown












            Required, but never shown







            Required, but never shown







            Popular posts from this blog

            As a Security Precaution, the user account has been locked The Next CEO of Stack OverflowMS...

            Список ссавців Італії Природоохоронні статуси | Список |...

            Українські прізвища Зміст Історичні відомості |...